LSAT and Law School Admissions Forum

Get expert LSAT preparation and law school admissions advice from PowerScore Test Preparation.

User avatar
 Dave Killoran
PowerScore Staff
  • PowerScore Staff
  • Posts: 5853
  • Joined: Mar 25, 2011
|
#80538
This game is also discussed in our Podcast: LSAT Podcast Episode 70: The May 2020 LSAT-Flex Logic Games Section

Complete Question Explanation
(The complete setup for this game can be found here: lsat/viewtopic.php?t=33051)

The correct answer choice is (D).

If H hosts a meeting in the fall, then the first rule is enacted and M must also hold a meeting in the fall:

  • ..... ..... _____ ..... ..... .....      _____

    ..... ..... __H__ ..... ..... .....       _____

    ..... ..... __M__ ..... ..... ..... _V/M_
    ..... ..... Fall ..... ..... .....      Spring
This also satisfies the third rule as far as the fall semester, but it leaves open a number of possible outcomes for the remaining spaces.

When applying the first obviously linked rule in a game does not eliminate any answers, leaves you with a wide open set of possibilities, and leaves you with no obvious next step with the rules or inferences, don't forget to check prior work! In this case, both questions #13, #14, and #16 featured solutions where H hosted in the fall. Using those solutions, you could eliminate answer choice (A) (via #14E or #16), answer choice (B) (via #13 or #16), and answer choice (C) (via #14E). At that point, you'd be down to answers (D) and (E), and since this is a Must Be True question it would be easy to try one of the two to see if it had to be the case. If it did, it's correct; if not, the other answer is correct.

If you did not think to use prior work, then the other avenue is to test out some hypotheticals. In this case, there are only three options, each based on the third and final city to host in the fall:

Solution #1: O hosts in the fall

When O hosts in the fall, then T and V must host in the spring. This satisfies both the second and third rules, leaving the choice H, M, or O to host in the spring:

  • ..... ..... _O_ ..... ..... .....      _H/M/O_

    ..... ..... __H__ ..... ..... ..... __T__

    ..... ..... __M__ ..... ..... ..... __V__
    ..... ..... Fall ..... ..... .....      Spring
This solution can be used to eliminate answer choices (B) and (C).



Solution #2: T hosts in the fall

When T hosts in the fall, then O and V must host in the spring. And, since V is hosting in the spring, the second rule applies and T must also host in the spring:

  • ..... ..... _T_ ..... ..... .....      __T__

    ..... ..... __H__ ..... ..... ..... __O__

    ..... ..... __M__ ..... ..... ..... _V_
    ..... ..... Fall ..... ..... .....      Spring
This solution can be used to eliminate answer choices (A) and (B).


Solution #3: V hosts in the fall

When V hosts in the fall, then O and T must host in the spring. The only remaining question is whether M or V then hosts in the spring as well:

  • ..... ..... _V_ ..... ..... .....      _____

    ..... ..... __H__ ..... ..... .....       _____

    ..... ..... __M__ ..... ..... ..... _M/V_
    ..... ..... Fall ..... ..... .....      Spring

This solution can be used to eliminate answer choices (A), (B), and (E).

Combining all of the information above, all answers except (D) have been eliminated.


Answer choice (A): As shown in solutions #2 and #3, O does not have to host a meeting in the fall.

Answer choice (B): As shown in solutions #1, #2, and #3, M does not have to host a meeting in the spring.

Answer choice (C): As shown in solution #1, O does not have to host a meeting in the spring.

Answer choice (D): This is the correct answer choice.

Answer choice (E): As shown in solution #3, V does not have to host a meeting in the spring.
User avatar
 rbrar_71
  • Posts: 3
  • Joined: Nov 02, 2021
|
#92929
Hey Dave,

After making the initial inference that both H & M will occur in the fall, can you just use the contrapositive of the second rule to solve this problem? In this case, if T is not in the spring (it would have to be in fall now), then V would also not occur in the spring. Seeing as there is only space for one variable in the fall now, the case is that T must occur in the spring. Let me know if my thought process here is correct, thanks!
 Rachael Wilkenfeld
PowerScore Staff
  • PowerScore Staff
  • Posts: 1358
  • Joined: Dec 15, 2011
|
#92935
Absolutely rbrar! That's a great way to look at this one too.

And yet another way to look at this one is to start by putting either M or V in the spring, since we know at least one of the two has to be there. We can only use one of the cities twice because we must use all five cities across the six slots.

Option 1: M is in the spring

Fall: M, H
Spring M

If M is in the spring, M would be our city used twice. That means every other city could only be used once. V could go in the fall, with T and O finishing spring. Or V could go in the spring, requiring T in the spring and pushing O to the fall.

Option 2: V is in the spring
Fall: M, H
Spring: V, T

In this case, there's a lot of certainty. But notice that T is in the spring. Just as it was in both scenarios under option 1. We've exhausted the possibilities.

The contrapositive of rule 2, as you have pointed out, is a fast way of seeing the same thing. T must be in the spring.

Great work!

Get the most out of your LSAT Prep Plus subscription.

Analyze and track your performance with our Testing and Analytics Package.